Kysymys # 82567

Kysymys # 82567
Anonim

Vastaus:

#cos ((2pi) / 9) + isin ((2pi) / 9) #, #cos ((8pi) / 9) + isin ((8pi) / 9) # ja

#cos ((14pi) / 9) + isin ((14pi) / 9) #,

Selitys:

Ensimmäinen asia on asettaa numero muotoon # Rhoe ^ (thetai) #

# Rho = sqrt ((1/2) ^ 2 + (sqrt (3) / 2) ^ 2) = sqrt (1/4 + 3/4) = 1 #

# Theta = arctan ((sqrt (3) / 2) / (- 1/2)) = arctan (-sqrt (3)) = - pi / 3 + kpi #. Valitaan # (2pi) / 3 #koska olemme toisessa neljänneksessä. Kiinnitä huomiota # Pi / 3 # on neljännellä neljänneksellä, ja tämä on väärin.

Numerosi on nyt:

# 1e ^ ((2pii) / 3) #

Nyt juuret ovat:

#root (3) (1) e ^ (((2kpi + (2pi) / 3) i) / 3), k ZZ #

# = e ^ ((((6kpi + 2pi) i) / 9), k ZZ #

joten voit valita k = 0, 1, 2 ja saada:

#E ^ ((2pii) / 9 #, #E ^ ((8kpii) / 9 # ja #E ^ ((14kpii) / 9 #

tai #cos ((2pi) / 9) + isin ((2pi) / 9) #, #cos ((8pi) / 9) + isin ((8pi) / 9) # ja

#cos ((14pi) / 9) + isin ((14pi) / 9) #.

Minulle tämä on umpikuja, koska en voi laskea trigonometrisiä funktioita # Pi / 9 #. Meidän on luotettava laskimeen:

# 0,7660 + 0.6428i #

# -0,9397 + 0.3420i #

# 0.1736-0.9848i #